Physik III, WS 1992/93
Lösungen zu den Übungen Nr.13
Besprechung: 10.Februar 1993
Aufgabe 1: (Bonusaufgabe) (6 Punkte)
a) Die Intensitätsverteilung am unendlich langen Einzelspalt ist nach Vorlesung

\begin{displaymath}
I = I_{0} \frac{sin^{2}\beta}{\beta^{2}}
\end{displaymath}

mit $\beta = (kb/2) sin\theta = (\pi b/\lambda) sin\theta$. Extrema ergeben sich bei

\begin{displaymath}
\frac{dI}{d\beta} = I_{0} \frac{2 \beta^{2} sin\beta \; cos...
...c{2 sin\beta
(\beta \; cos\beta - sin\beta)}{\beta^{3}} = 0
\end{displaymath}

Minima ergeben sich für $sin\beta =0$ und $\beta \neq 0$, d.h.

\begin{displaymath}
\beta_{min,m} = \pm m \pi, \; \; \; \; \; m=1,2,3,...
\end{displaymath}

Maxima erhält man entsprechend für $\beta \; cos\beta - sin\beta = 0$ oder $\beta = tg(\beta)$. Letzteres wird erfüllt durch:

\begin{displaymath}
\beta_{max,1} \approx \pm 1,43 \pi \; \; \; \; \; \; \; \;
...
... \; \; \; \; \; \; \; \;
\beta_{max,3} \approx \pm 3,47 \pi
\end{displaymath}

Die Maxima liegen also nicht exakt zwischen den Minima, sondern geringfügig zu kleineren Winkeln verschoben. Wegen

\begin{displaymath}
\theta \approx sin\theta = \frac{2\beta}{kb} = \frac{\beta \lambda}
{\pi b}
\end{displaymath}

folgt für die Minima:

\begin{displaymath}
\theta_{min,m} = \pm m\pi \frac{\lambda}{\pi b} =
\underline{\pm m \; 5\cdot 10^{-4}}, \; \; \; \; \; m=1,2,3,...
\end{displaymath}

Für die ersten drei Nebenmaxima gilt:

\begin{displaymath}
\theta_{max,1} \approx \pm 7,14 \cdot 10^{-4} \; \; \; \; \...
...; \; \; \; \;
\theta_{max,3} \approx \pm 1,73 \cdot 10^{-3}
\end{displaymath}

b) Näherungsweise liegen die Maxima zwischen den Minima, d.h. bei

\begin{displaymath}
\beta_{max,m} \approx \pm (m+\frac{1}{2}) \pi, \; \; \; \; \; \; \;
m=1,2,3,...
\end{displaymath}

Dann ist aber $sin\beta_{max,m} = 1$ und daher

\begin{displaymath}
I_{m} = I_{0} \frac{1}{\beta^{2}} = I_{0} \frac{1}{[(m+1/2)\pi]^{2}}
\end{displaymath}

c) Die Intensität fällt auf die Hälfte ihres Wertes ab, wenn

\begin{displaymath}
\frac{I(\theta)}{I_{0}} = \frac{sin^{2}\beta}{\beta^{2}} = \frac{1}{2}
\end{displaymath}

oder $sin\beta = 0,707 \; \beta$. Diese Gleichung wird gelöst durch $\beta_{1/2} \approx 1,39$. Wegen $\beta = (\pi b/\lambda) sin\theta \approx (\pi b/\lambda ) \theta$ folgt:

\begin{displaymath}
\theta_{1/2} = \frac{\lambda}{\pi b} \beta_{1/2}
\end{displaymath}

und

\begin{displaymath}
\Delta \theta_{1/2} = 2 \theta_{1/2} =
0,885 \frac{\lambda}{b} \approx \underline{\frac{\lambda}{b}}
\end{displaymath}

Aufgabe 2: (Bonusaufgabe) (6 Punkte)
Die Berechnung der Intensitätsverteilung bei der Beugung an ausgedehnten Strukturen ist vom Prinzip her sehr einfach, die Komplikation liegt häufig einzig und allein in der Berechnung komplizierter Integrale. Wir diskutieren zunächst noch einmal ausführlich die Berechnung der Intensitätsverteilung am unendlich langen Spalt und erweitern das Ergebnis danach auf den Doppelspalt. Eine ebene Lichtwelle der Wellenlänge $\lambda$ falle senkrecht auf einen unendlich langen Spalt der Breite $b$.

Jedes durch $dx$ charakterisierte Flächenelement des Spaltes ist nach Huygens Zentrum einer Kugelwelle. Die gesamte Lichterregung im weit entfernten, durch den Beugungswinkel $\phi$ charakterisierten Aufpunkt $P$ wird durch Integration über alle Elemente $dx$ des Spaltes erhalten, wobei die verschiedenen Phasen der Elementarwellen zu berücksichtigen sind. Wenn wir für die Größe der von $dx$ ausgehenden Wellenamplitude den Ansatz

\begin{displaymath}
dE_{0} = E_{0} \; \frac{dx}{b}
\end{displaymath}

machen, dann ist die Elementarwelle im Punkt $P$ darstellbar durch

\begin{displaymath}
dE = dE_{0} \; \frac{cos(\omega t - kr)}{r} = E_{0} \; \frac{dx}{b} \;
\frac{cos(\omega t - kr)}{r}
\end{displaymath}

Hierbei ist allerdings bereits die Annahme eingeführt, daß alle infinitesimalem Beiträge zur Welle im Punkt $P$ parallel sind. Wegen des Vektorcharakters der Feldstärke $\vec{E}$ ist dieses eine Näherung, die nur für $b \ll r$ erfüllt ist. Ohne diese Näherung werden die nachfolgend zu berechnenden Integrale außerordentlich kompliziert. Mit der bereits eingeführten Näherung $b \ll r$ ist auch $r \approx r_{0} - x \; sin\phi$, daher

\begin{displaymath}
dE \approx E_{0} \; \frac{dx}{b}
\; \frac{cos(\omega t -kr_{0} + kx \; sin\phi)}{r_{0}}
\end{displaymath}

Diese Formel beendet den physikalischen Teil der Aufgabe, es folgt der mathematische Teil. Die Gesamtamplitude im Aufpunkt $P$ ergibt sich aus der Summe (Integral) aller Teilamplituden der verschiedenen Flächenelemente $dx$. Dieses Integration kann auf der reellen Zahlengerade oder in der komplexen Ebene durchgeführt werden. Wir zeigen zunächst die reelle Integration:

\begin{displaymath}
E(r_{0},\phi) = \frac{E_{0}}{r_{0}b} \int_{0}^{b} dx \; cos(\omega t - kr_{0}
+ kx \; sin\phi)
\end{displaymath}

Dieses Integral wird gelöst durch
\begin{displaymath}
E(r_{0},\phi) = \frac{E_{0}}{r_{0}b} \left[ \frac{sin(\omeg...
...r_{0} +kb \sin\phi)
- sin(\omega t - kr_{0})}{kb \; sin\phi}
\end{displaymath} (1)

Mit Hilfe trigonometrischer Formeln kann man zeigen, daß

\begin{displaymath}
E(r_{0},\phi) = \frac{E_{0}}{r_{0}} \; \frac{sin[(kb\; sin\...
...kb\; sin\phi)/2} \; cos[\omega t - kr_{0} + (kb\; sin\phi)/2)
\end{displaymath}

Die Integration und die nachfolgenen Umrechnungen kann man vereinfachen, wenn man die Feldstärke als Realteil einer komplexen Funktion schreibt.

\begin{displaymath}
E(r_{0},\phi) = \frac{E_{0}}{r_{0}b} \int_{0}^{b} dx \;
e...
...{i(\omega t - kr_{0})} \int_{0}^{b} dx \; e^{ikx \; sin\phi}
\end{displaymath}

Dieses Integral ist leicht auszuwerten und ergibt
\begin{displaymath}
E(r_{0},\phi) = \frac{E_{0}}{r_{0}b} e^{i(\omega t -kr_{0})...
... \left(
\frac{e^{i(kb \; sin\phi)}-1}{ik \; sin\phi} \right)
\end{displaymath} (2)


\begin{displaymath}
E(r_{0},\phi) = \frac{E_{0}}{r_{0}b} e^{i(\omega t -kr_{0})...
...\phi)/2}}
{2ik \; sin\phi} \right) 2 e^{i(kb \; sin\phi)/2}
\end{displaymath} (3)

Wegen $(e^{iz}-e^{-iz})/(2i) = sin(z)$ folgt:

\begin{displaymath}
E(r_{0},\phi) = \frac{E_{0}}{r_{0}} \; \frac{sin[(kb \; sin...
...\; sin\phi)/2 } \; e^{i(\omega t -kr_{0} +(kb \; sin\phi)/2)}
\end{displaymath}

Wir gehen zurück zum Realteil und erhalten:

\begin{displaymath}
E(r_{0},\phi) = \frac{E_{0}}{r_{0}} \frac{sin[(kb \; sin\ph...
...(kb \; sin\phi)/2} cos[\omega t -kr_{0} + (kb \; sin\phi)/2]
\end{displaymath}

also das gleiche Ergebnis wie oben. Die Intensität ist proportional $E^{2}$, gemittelt über die Zeit $t$. Da allgemein

\begin{displaymath}
\overline{cos^{2}(\omega t + \alpha)} = \frac{1}{2}
\end{displaymath}

folgt in unserem Beispiel

\begin{displaymath}
I \sim \overline{E^{2}} = \frac{E_{0}^{2}}{r_{0}^{2}} \;
...
...\;
\frac{sin^{2}[(kb\; sin\phi)/2]}{[(kb\; sin\phi)/2]^{2}}
\end{displaymath}

Wegen $k = 2\pi/\lambda$ kann man dieses auch schreiben als:

\begin{displaymath}
I = I_{0} \;
\frac{sin^{2}[(\pi b \; sin\phi)/\lambda]}{[\pi b \; sin\phi)/\lambda]^{2}}
\end{displaymath}

Die Erweiterung auf den Doppelspalt ist nun denkbar einfach, man muß die Integration nur über beide Spalte erstrecken:


\begin{displaymath}
E(r_{0},\phi)= \frac{E_{0}}{r_{0}b} e^{i(\omega t - kr_{0})...
...phi} + \int_{a-b/2}^{a+b/2}
dx \; e^{ikx \; sin\phi} \right)
\end{displaymath} (4)

Die Integration wird exakt wie beim Einzelspalt durchgeführt, nur mit etwas mehr Schreibarbeit. Das Ergebnis ist:

\begin{displaymath}
I = I_{0} \frac{sin^{2}\beta}{\beta^{2}} cos^{2}\alpha
\end{displaymath}

mit $\beta = (\pi b/\lambda) sin\phi$ und $\alpha = (\pi a/\lambda) sin\phi$.
b) Skizze:


Aufgabe 3: (Bonusaufgabe) (4 Punkte)
Die Beugung an einer kreisförmigen Öffnung ergibt ein System konzentrischer Maxima und Minima. Nach Vorlesung erscheint das erste Minimum unter dem Winkel $\phi_{1} \approx 1,22 \lambda/d$, wobei $d$ der Durchmesser der Öffnung ist. Daher folgt für den Radius des Minimums auf der Netzhaut

\begin{displaymath}
R = f \alpha = 1,22 \frac{f \lambda'}{d}
\end{displaymath}

mit $\lambda' = \lambda/n$. Einsetzen der Zahlenwerte: $R = 5,19 \; \lambda$. Setzt man eine mittlere Wellenlänge von $550 \; nm$ ein, so folgt $R= 2,85 \; \mu m$ und für den Durchmesser $\underline{D \approx 5,7 \; \mu m}$.
Aufgabe 4: (Bonusaufgabe) (4 Punkte)
Wegen $d = \lambda/(n sin\alpha)$ und $n=1$, $\alpha = 60^{o}$ folgt $\underline{d = 680 \; nm}$.



Harm Fesefeldt
2007-08-24